49. [L03] Palese?

Tutti i problemi che presentino una figura (calcolo delle aree e dei perimetri, similitudini, allineamenti, concorrenze, ecc...)
Rispondi
Gerald Lambeau
Messaggi: 920
Iscritto il: 07/01/2015, 18:18

49. [L03] Palese?

Messaggio da Gerald Lambeau »

Siano $\Gamma_1, \Gamma_2, \Gamma_3$ tre circonferenze concorrenti in $P$. Sia $Q$ l'intersezione, diversa da $P$, di $\Gamma_1$ e $\Gamma_2$ e $R$ l'intersezione, diversa da $P$, di $\Gamma_1$ e $\Gamma_3$. Sia $A$ un punto su $\Gamma_1$ e siano $B$ il punto d'intersezione fra la retta $AQ$ e $\Gamma_2$ diverso da $Q$ e $C$ il punto d'intersezione fra la retta $AR$ e $\Gamma_3$ diverso da $R$. Dimostrare che, al variare di $A$ su $\Gamma_1$, le rette $BC$ concorrono.
"I matematici non realizzano nulla... semplicemente scoprono e dimostrano verità intrinseche riguardanti tutto ciò che esiste, ovvietà e banalità per una mente superiore, perfetta. Ed è quello il mio obiettivo!"
Cit. Marco (mio vero nome)
Avatar utente
Giovanni98
Messaggi: 1255
Iscritto il: 27/11/2014, 14:30

Re: 49. [L03] Palese?

Messaggio da Giovanni98 »

EDIT : .
Ultima modifica di Giovanni98 il 17/05/2016, 19:06, modificato 1 volta in totale.
Gerald Lambeau
Messaggi: 920
Iscritto il: 07/01/2015, 18:18

Re: 49. [L03] Palese?

Messaggio da Gerald Lambeau »

Giovanni98 ha scritto:Per prima cosa notiamo che dalla ciclicità di $APQR$ si ha $ARP = AQP$.
:?: No! E se fossero opposti? Poi $CRP=ARP$ mi sembra proprio errata... almeno per come ho fatto io il disegno; diciamo che in questo problema ci sono un bel po' di casi da fare.
"I matematici non realizzano nulla... semplicemente scoprono e dimostrano verità intrinseche riguardanti tutto ciò che esiste, ovvietà e banalità per una mente superiore, perfetta. Ed è quello il mio obiettivo!"
Cit. Marco (mio vero nome)
Avatar utente
Giovanni98
Messaggi: 1255
Iscritto il: 27/11/2014, 14:30

Re: 49. [L03] Palese?

Messaggio da Giovanni98 »

Si, me ne sono accorto solo ora.
pipotoninoster
Messaggi: 19
Iscritto il: 23/02/2018, 12:08

Re: 49. [L03] Palese?

Messaggio da pipotoninoster »

Ci provo...
Testo nascosto:
Lemma: Sia [tex]UVW[/tex] un triangolo. Sia [tex]K_U[/tex] un punto sulla retta [tex]VW[/tex] e cicliche. Allora le circoscritte ai triangoli [tex]K_VUK_W[/tex], [tex]K_WVK_U[/tex], [tex]K_UWK_V[/tex] concorrono. Per dimostrarlo si devono considerare 4 casi:
-tutti i [tex]K[/tex] sono interni ai lati.
-due sono interni, uno è esterno.
-uno è interno, due sono esterni.
-tutti e tre sono esterni.
In ognuno dei casi si procede con angle chasing,
Per esempio, nel primo caso, se [tex]\angle WUV=u[/tex] e cicliche, sia [tex]P[/tex] l'intersezione tra le circoscritte a [tex]K_WVK_U[/tex], [tex]K_UWK_V[/tex]. Allora [tex]\angle K_WPK_U=\pi-v[/tex], [tex]\angle K_UPK_V=\pi-w[/tex]. E [tex]\angle K_VPK_W=\pi-u[/tex], cioè [tex]K_VUK_WP[/tex] è cicilico, cioè lemma.
Ora, sia [tex]T[/tex] la seconda intersezione tra [tex]\Gamma_2[/tex] e [tex]\Gamma_3[/tex]. Dimostriamo che [tex]T \in BC[/tex]. Supponiamo per assurdo che [tex]BC \cap \Gamma_2= \left \{ T' \right \}[/tex], [tex]T \ne T'[/tex]. Allora per il lemma su [tex]ABC[/tex] e [tex]Q,R,T'[/tex] si ha che [tex]P[/tex] sta sulla circoscritta a [tex]CRT'[/tex], cioè [tex]T'[/tex] sta sulla circoscritta a [tex]CRP[/tex] che è [tex]\Gamma_3[/tex]. Ora, abbiamo che:
[tex]T' \in \Gamma_2[/tex] e [tex]T' \in \Gamma_3[/tex], ossia [tex]T'[/tex] coincide con [tex]T[/tex] oppure con [tex]P[/tex]. Se fosse [tex]T'=P[/tex] significa
che [tex]\Gamma_2[/tex] e [tex]\Gamma_3[/tex] sono tangenti, cioè [tex]T'=P=T[/tex]. In ogni caso [tex]BC[/tex] passa per [tex]T[/tex] al variare di [tex]A[/tex]
Rispondi